Đến nội dung

Hình ảnh

$\frac{a_1}{a_{2}+a_3+...a_n+1}+\frac{a_2}{a_1+a_3+...+a_n+1}+...+\frac{a_n}{a_1+a_2+...+a_{n-1}} + (1-a_1)(1-a_2)...(1-a_n)\leq 1$.


  • Please log in to reply
Chủ đề này có 1 trả lời

#1
Murasaki Yasu

Murasaki Yasu

    Binh nhì

  • Thành viên mới
  • 16 Bài viết

Cho $a_{1}, a_{2},...,a_{n}\epsilon [0,1]$.

Chứng minh:$\frac{a_1}{a_{2}+a_3+...a_n+1}+\frac{a_2}{a_1+a_3+...+a_n+1}+...+\frac{a_n}{a_1+a_2+...+a_{n-1}} + (1-a_1)(1-a_2)...(1-a_n)\leq 1$.



#2
cristianoronaldo

cristianoronaldo

    Thượng sĩ

  • Thành viên
  • 233 Bài viết

 

Cho $a_{1}, a_{2},...,a_{n}\epsilon [0,1]$.

Chứng minh:$\frac{a_1}{a_{2}+a_3+...a_n+1}+\frac{a_2}{a_1+a_3+...+a_n+1}+...+\frac{a_n}{a_1+a_2+...+a_{n-1}} + (1-a_1)(1-a_2)...(1-a_n)\leq 1$.

 

Không mất tính tổng quát, giả sử $a_{1}\geq a_{2}\geq ...\geq a_{n}$. Khi đó, ta có:

$VT\leq \frac{a_{1}}{a_{2}+a_{3}+...+a_{n}+1}+\frac{a_{2}}{a_{2}+a_{3}+...+a_{n}+1}+...+\frac{a_{n}}{a_{2}+a_{3}+...+a_{n}+1}=\frac{a_{1}+...+a_{n}}{1+a_{2}+...+a_{n}}=1-\frac{1-a_{1}}{1+a_{2}+...+a_{n}}$

Như vậy, ta cần chứng minh:

$(1-a_1)(1-a_2)...(1-a_n)-\frac{1-a_{1}}{a_{2}+a_{3}+...+a_{n}+1}\leq 0$

$\Leftrightarrow (1-a_2)...(1-a_n)(1+a_{2}+a_{3}+...+a_{n})\leq 1$ ( luôn đúng theo AM-GM)

$\Rightarrow Q.E.D$


Bài viết đã được chỉnh sửa nội dung bởi cristianoronaldo: 24-07-2017 - 16:54

Nothing in your eyes





0 người đang xem chủ đề

0 thành viên, 0 khách, 0 thành viên ẩn danh